标量场的梯度

\[\begin{align*} \newcommand{\dif}{\mathop{}\!\mathrm{d}} \newcommand{\p}{\partial} \end{align*}\]

标量场的等值面

标量场
标量场在空间中的分布规律可以用一个标量函数来表示,标量函数必须是连续可微函数
等值面
等值面就是使标量场取相同值得点所构成得空间曲面
对于任意给定的常数 C,方程 u=u(x, y, z)=C 就是等值面方程。每个 C 都对应唯一的曲面与其对应
场中的等值面互不相交(否则不满足连续可微)

方向导数

方向导数
设 $M_0$ 为标量场 $u=u(M)$ 中的一点,从点 $M_0$ 出发引出一条射线 $l$,在 $l$ 上临近点 $M_0$ 处取一动点 $M$,记$\Delta l = MM_0$,则极限:
\[\lim_{\Delta l \rightarrow 0} \frac{u(M) - u(M_0)}{\Delta l} = \frac{\partial u}{\partial l} \Big\vert_{M_0}\]
称为标量场 $u(M)$ 在点 $M_0$ 处沿 $l$ 方向上的方向导数

计算公式

设标量场在点 $M_0(x_0, y_0, z_0)$ 处可微,$M(x_0+\Delta x, y_0+\Delta y, z_0+\Delta z)$,则:

\[u(M) - u(M_0) = \Delta u = \frac{\partial u}{\partial x}\Delta x + \frac{\partial u}{\partial y}\Delta y+ \frac{\partial u}{\partial z}\Delta z\]

而:

\[\Delta x = \Delta l \cos \alpha,\; \Delta y = \Delta l \cos \beta,\; \Delta z = \Delta l \cos \gamma\]

从而

\[\frac{\partial u}{\partial l} = \frac{\partial u}{\partial x} \cos \beta + \frac{\partial u}{\partial y} \cos \alpha+ \frac{\partial u}{\partial z} \cos \gamma\]

进一步我们设坐标单位矢量为 $( \hat{a}_x, \hat{a}_y, \hat{a}_z )$,$\vec{l}$ 方向的单位矢量为 $\hat{a}_l$,把 $\cos$ 化成:

\[\frac{\partial u}{\partial l} = \Big( \frac{\partial u}{\partial x} \hat{a}_x + \frac{\partial u}{\partial y} \hat{a}_y+ \frac{\partial u}{\partial z} \hat{a}_z \Big) \cdot \hat{a}_l\]

梯度矢量

\[\text{令}\; \vec{G} = \frac{\partial u}{\partial x} \hat{a}_x + \frac{\partial u}{\partial y} \hat{a}_y+ \frac{\partial u}{\partial z} \hat{a}_z\\ \text{则}\; \frac{\partial u}{\partial l} = \vec{G} \cdot \hat{a}_l = \vert \vec{G} \vert \cos<\vec{G}, \hat{a}_l>\]

当 $\vec{l}$ 与 $\vec{G}$ 方向一致时,标量场在点 $M$ 处方向导数最大(此时 $\frac{\partial u}{\partial l} = \vert \vec{G} \vert$),矢量函数 $\vec{G}$ 定义为 梯度,记为 $\vec{G} = \text{grad} u$ 或 $\nabla u$,我们称 $\nabla$ 为那勃勒算子(nabla)或哈密顿算子

在电场中,梯度具有如下性质:

  • 与等电位面垂直
  • 数值上等于最大方向导数
  • 指向点位增加的方向

下面列出各坐标系中,梯度的表达式:

直角坐标系 \(\text{grad}u = \frac{\partial u}{\partial x} \hat{a}_x + \frac{\partial u}{\partial y} \hat{a}_y+ \frac{\partial u}{\partial z} \hat{a}_z\)

柱坐标系$\text{grad} u = \frac{\partial u}{\partial r} \vec{a}_r + \frac{1}{r}\frac{\partial u}{\partial \varphi} \vec{a}_\varphi + \frac{\partial u}{\partial z} \vec{a}_z$

以下是推导过程:

\[\begin{align*} \dif \vec{l} &= \hat{a}_r \dif l_r + \hat{a}_\varphi r \dif \varphi + \hat{a}_z \dif z\\\\ \dif u &= \frac{\partial u}{\partial r} \dif r + \frac{\partial u}{\partial \varphi} \dif \varphi + \frac{\partial u}{\partial z} \dif z\\ &= \frac{\partial u}{\partial r} \dif r + \frac{1}{r}\frac{\partial u}{\partial \varphi} r\dif \varphi + \frac{\partial u}{\partial z} \dif z\\ &= \Big( \frac{\partial u}{\partial r} \vec{a}_r + \frac{1}{r}\frac{\partial u}{\partial \varphi} \vec{a}_\varphi + \frac{\partial u}{\partial z} \vec{a}_z \Big) \cdot \dif \vec{l}\\ &= \text{grad} u \cdot \dif \vec{l} \end{align*}\]

球坐标系 $\text{grad} u = \frac{\partial u}{\partial R} \vec{a}_R + \frac{1}{R}\frac{\partial u}{\partial \theta} \vec{a}_\theta + \frac{1}{R \sin \theta} \frac{\partial u}{\partial \varphi} \vec{a}_\varphi$

推导过程类似于柱坐标系

注意

请一定要记住各个坐标系的 $\nabla$算子:

  1. 直角:$\frac{\partial u}{\partial x}+\frac{\partial u}{\partial y}+\frac{\partial u}{\partial z}$
  2. 柱:$\frac{\partial u}{\partial r}+\frac{1}{r}\frac{\partial u}{\partial \varphi}+\frac{\partial u}{\partial z}$
  3. 球:$\frac{\partial u}{\partial R}+\frac{1}{R}\frac{\partial u}{\partial \theta}+\frac{1}{R \sin \theta} \frac{\partial u}{\partial \varphi}$

而实际上上面都是由 $\dif \vec{l}$ 推导出来的。所以别看第一章第一节很简单,其实是很难的。

梯度的性质:

  • $\nabla C = 0$
  • $\nabla (u\pm v) = \nabla u \pm \nabla v$
  • $\nabla(uv)=u\nabla v+v \nabla u$
  • $\nabla\left( \dfrac{u}{v} \right)=\frac{1}{v^2}(v\nabla u-u \nabla v)$
  • $\nabla f(u) = f’(u)\nabla u$

可以看出,梯度的性质和求导是一模一样的。


从数学上我们知道,方向导数可以用梯度表示为:

\[\frac{\partial u}{\partial l} = \hat{a}_l \cdot\text{grad} u\]

(更严谨的推导请参考数学分析

题目:$R=\sqrt{(x-x’)^2+(y-y’)^2+(z-z’)^2}$,试证明 $\nabla (\frac{1}{R})=-\nabla’ (\frac{1}{R})$。其中,$(x’,y’,z’)$ 是一定点,$\nabla’$ 表示对 $x’,y’,z’$ 微分。

这题看上去应该用球坐标系,但实际用直角会更简单:

\[\begin{align} \frac{\p}{\p x} \left( \frac{1}{R} \right)&=\frac{\p R}{\p x}\frac{\p}{\p R} \left( \frac{1}{R} \right)\\ &=-\frac{1}{R^2}\cdot \frac{2(x-x')}{2\sqrt{(x-x')^2+(y-y')^2+(z-z')^2}}\\ &=-\frac{1}{R^3}(x-x')\\ \therefore \nabla \left( \frac{1}{R} \right)&=-\frac{1}{R^3} \left[ (x-x')\vec{a}_x+(y-y')\vec{a}_y+(z-z')\vec{a}_z \right]\\ &=-\frac{\vec{R}}{R^3} \end{align}\] \[\begin{align} \frac{\p}{\p x'} \left( \frac{1}{R} \right)&=\frac{\p R}{\p x'}\frac{\p}{\p R} \left( \frac{1}{R} \right)\\ &=\frac{1}{R^2}\cdot \frac{2(x-x')}{2\sqrt{(x-x')^2+(y-y')^2+(z-z')^2}}\\ &=\frac{1}{R^3}(x-x')\\ \therefore \nabla \left( \frac{1}{R} \right)&=\frac{1}{R^3} \left[ (x-x')\vec{a}_x+(y-y')\vec{a}_y+(z-z')\vec{a}_z \right]\\ &=\frac{\vec{R}}{R^3} \end{align}\]

综上,$\nabla (\frac{1}{R})=-\frac{\vec{R}}{R^3}=-\nabla’ (\frac{1}{R})$. 这个结论以后会经常用到。